Cardiology Flashcards
A 45 year-old man developed retrosternal chest pain during a board meeting. A raised blood pressure had been noted during a medical three years previously, but no treatment had been initiated. What is the most likely diagnosis on ECG?
A ) Anterior MI
B ) Anteroseptal MI
C ) Complete heart block
D ) Hyperkalemia
E ) Inferior MI
F ) Inferolateral MI
G ) Lateral MI
H ) Mobitz Type II AV block
I ) Pericarditis
J ) Supraventricular tachycardia (SVT)
K ) Ventricular tachycardia
L ) Wolff-Parkinson-White syndrome
F ) Inferolateral MI
Which of the following conditions is associated with radio-radial delay?
A ) Aortic dissection
B ) Mitral regurgitation
C ) Spontaneous pneumothorax
D ) Tricuspid stenosis
E ) Ventricular septal defect
A ) Aortic dissection
Which of the following features is consistent with aortic stenosis?
- A ) Begins after the isovolumetric contraction
- B ) Often associated with a pistol shot pulse
- C ) Often associated with AF
- D ) Radiates to the axilla
Answer: Begins after the isovolumetric contraction
Which of the following is a cause of sinus bradycardia?
A ) Anaemia
B ) Beta-agonists
C ) Fever
D ) Raised intracranial pressure
E ) Thyrotoxicosis
D ) Raised intracranial pressure
Which valvular defect is almost exclusively caused by rheumatic heart disease?
A ) Aortic regurgitation
B ) Mitral regurgitation
C ) Mitral stenosis
D ) Tricuspid stenosis
E ) Ventricular septal defect
C ) Mitral stenosis
Atrial fibrillation that fails to self-terminate within 7 days is classed as:
A ) Lone AF
B ) Paroxysmal AF
C ) Permanent AF
D ) Persistent AF
E ) Subclinical AF
D ) Persistent AF
A 50-year-old woman presents to her GP with a 3-month history of pain in the right buttock. The pain is cramping in nature and only comes after a few minutes of walking. The pain radiates to the right thigh and stops when the patient stops walking. Which one of the following is the most likely explanation?
A ) L4/L5 disc prolapse
B ) OA of the right hip
C ) Spinal canal stenosis
D ) Stenosis of right common iliac artery
E ) Stenosis of the right superficial femoral artery
D ) Stenosis of right common iliac artery
Answer: Atropine
A ) First-degree AV block
B ) Inferior-posterior-lateral STEMI
C ) Isolated inferior STEMI
D ) Isolated lateral STEMI
E ) Left bundle branch block (LBBB)
F ) Pericarditis
G ) Premature ventricular contraction
H ) Right bundle branch block (RBBB)
I ) Second-degree AV block (Type I)
J ) Second-degree AV block (Type II)
K ) Third-degree AV block
L ) Ventricular tachycardia
F ) Pericarditis
How many classical stages of congestive heart failure exist?
A ) 1
B ) 2
C ) 3
D ) 4
E ) 5
D) 4
An 73-year-old man presents to ED with a 3-day history of shortness of breath on exertion. On examination you note an irregularly irregular pulse, and ECG confirms atrial fibrillation.
As part of your work-up, which scoring system will help you assess ischaemic stroke risk in this patient?
A ) CHADS-VASc score
B ) HAS-BLED score
C ) PERC rule
D ) qSOFA
E ) The Wells’ Score
A) CHADS-VASc score
A ) Atrial fibrillation
B ) Atrial flutter
C ) Bundle branch block (BBB)
D ) First-degree AV block
E ) Paroxysmal supraventricular tachycardia
F ) Pericarditis
G ) Second-degree AV block (Type I)
H ) Second-degree AV block (Type II)
I ) Sinus tachycardia
J ) Third-degree AV block
I ) Sinus tachycardia
A ) 1st degree AV block
B ) 2nd degree (2:1 type) AV Block
C ) 2nd degree (Mobitz 1) AV Block
D ) 3rd degree AV block with LBBB
E ) 3rd degree AV block with RBBB
E) 3rd degree AV block with RBBB
A ) Hypercalcaemia
B ) Hyperkalaemia
C ) Hypernatraemia
D ) Hypocalcaemia
E ) Hypokalaemia
B ) Hyperkalaemia
A ) Bundle branch block
B ) First-degree AV block
C ) Hyperkalemia
D ) Pacemaker
E ) Paroxysmal supraventricular tachycardia
F ) Third-degree AV block
G ) Wolf-Parkinson-White syndrome
D) Pacemaker
A two-year-old male presents with fixed splitting of the second heart sound and an ejection systolic murmur in the pulmonary area. What is the most likely diagnosis?
A ) Aortic regurgitation
B ) Aortic stenosis
C ) Atrial septal defect
D ) Mitral regurgitation
E ) Mitral stenosis
F ) Patent ductus arteriosus
G ) Pulmonary stenosis
H ) Tricuspid regurgitation
I ) Ventricular septal defect
C ) Atrial septal defect
Which valvular defect is associated with a high-pitched, decrescendo, blowing diastolic murmur?
A ) Aortic regurgitation
B ) Aortic stenosis
C ) Pulmonary stenosis
D ) Tricuspid regurgitation
E ) Ventricular septal defect
A ) Aortic regurgitation
T-wave inversion is associated with all of the following pathologies, EXCEPT:
A ) Hyperkalaemia
B ) Hypokalaemia
C ) Left bundle branch block (LBBB)
D ) Myocardial infarction
E ) Pulmonary embolism
A ) Hyperkalaemia
Answer: Neurocardiogenic syncope
A ) Hyperkalaemia
B ) Hypokalaemia
C ) Inferior myocardial ischaemia
D ) Prinzmetal angina
E ) Raised ICP
C ) Inferior myocardial ischaemia
Answer: Coronary artery spasm
Which of the following is NOT part of the clinical management of vasospastic (also known as Printzmetal, or variant) angina?
A ) Avoid beta blockers
B ) Avoid precipitating factors such as smoking, and take GTN PRN
C ) Calcium channel blockers are a first-line agent
D ) Coronary angiogram and/or ergometrine challenge as indicated
E ) Potassium channel antagonists
E ) Potassium channel antagonists
A ) Circumflex
B ) Left anterior descending
C ) Left coronary artery
D ) Posterior interventricular artery
E ) Right coronary artery
** E ) Right coronary artery**
There is ST elevation in leads II, III, and aVF. Most inferior STEMIs (~80%) are associated with occlusion of a dominant right coronary artery.
C) 3/6: Accompanied by a parasternal heave
Answer: Mitral regurgitation
Answer: Wolff-Parkinson-White syndrome
Answer: Junctional rhythm
Answer: Previous MI
Answer: Low blood pressure, muffled heart sounds, raised JVP
Answer: Pulseless electrical activity
A ) Embolectomy
B ) Exercise programme
C ) Immediate vascular reconstruction
D ) Reassurance and simple analgesia
E ) Urgent vascular imaging
Answer: Urgent vascular imaging
Answer: Asystole
Answer: Aspirin
A ) 24 hour ECG monitoring
B ) BGL
C ) Coronary angiography
D ) Lipids
E ) TSH
Answer: TSH
Answer: ECG
Which one of the following is a side effect of beta-blockers?
A ) Reduce blood lipids
B ) Reduce sensitivity to hyperglycemia
C ) Tachycardia
D ) Tachypnea
E ) Vivid dreams
E ) Vivid dreams
A ) First-degree heart block
B ) Left bundle branch block
C ) Normal
D ) Sinus arrhythmia
E ) Wolff–Parkinson–White syndrome
Answer: Wolff–Parkinson–White syndrome
Answer: ACE Inhibitor
About 1-10% of patients will develop a dry, nonproductive paroxysmal cough on ACE-inhibitors.
Answer: Pericarditis
Answer: Beta-blocker
Answer: A third heart sound can be a sign of heart failure
A ) Beta-blockers are an ineffective treatment
B ) It can be caused by pneumonia
C ) It is often associated with hypothyroidism
D ) It is reversible with atropine
E ) It not a risk factor for stroke
Answer: It can be caused by pneumonia
This is atrial fibrillation. It can be idiopathic or precipitated by infection, stress, cardiovascular disease, electrolyte disturbance, and certain medications, amongst various other causes.